Difference between revisions of "1991 AHSME Problems/Problem 2"

(Created page with "<math>|3-\pi|=</math> (A) <math>\frac{1}{7}</math> (B) <math>0.14</math> (C) <math>3-\pi</math> (D) <math>3+\pi</math> (E) <math>\pi-3</math>")
 
Line 2: Line 2:
  
 
(A) <math>\frac{1}{7}</math>  (B) <math>0.14</math>  (C) <math>3-\pi</math>  (D) <math>3+\pi</math>  (E) <math>\pi-3</math>
 
(A) <math>\frac{1}{7}</math>  (B) <math>0.14</math>  (C) <math>3-\pi</math>  (D) <math>3+\pi</math>  (E) <math>\pi-3</math>
 +
{{MAA Notice}}

Revision as of 13:52, 5 July 2013

$|3-\pi|=$

(A) $\frac{1}{7}$ (B) $0.14$ (C) $3-\pi$ (D) $3+\pi$ (E) $\pi-3$ The problems on this page are copyrighted by the Mathematical Association of America's American Mathematics Competitions. AMC logo.png